Radiación de Hawking como M(t)→0M(t)→0M(t) \rightarrow 0

la luminosidad PAG de un agujero negro de Kerr-Newman con carga q y momento angular j es dado por

PAG = 1 240 C 6 ( 1 q 2 4 π ϵ 0 GRAMO METRO 2 ( j C METRO 2 GRAMO ) 2 ) 2 π GRAMO 2 METRO 2 ( 2 + 2 1 q 2 4 π ϵ 0 GRAMO METRO 2 ( j C METRO 2 GRAMO ) 2 q 2 4 π ϵ 0 GRAMO METRO ) 3 .
Para un agujero negro de Schwarzschild, establecemos q = 0 y j = 0 :
PAG = C 6 15360 π GRAMO 2 METRO 2 .
Además, a través de la equivalencia masa-energía, PAG da la tasa mi ˙ en el que la energía se irradia en partículas de masa metro (aquí suponemos que todas las partículas creadas tienen masa metro ):
PAG = d mi d t = d d t norte metro C 2 1 β 2 ,
dónde norte es el número de partículas creadas. Dado que norte A (y a la vez norte METRO 2 ), nosotros escribimos
norte = k A = dieciséis π k GRAMO 2 METRO 2 C 4 ,
dónde k es una constante de proporcionalidad arbitraria. Por eso,
PAG = dieciséis π k GRAMO 2 METRO 2 C 4 metro C 2 v ( 1 β 2 ) 3 / 2 d v d t C 8 245 760 π 2 k metro GRAMO 4 METRO 4 = v ( 1 β 2 ) 3 / 2 d v d t .
Es claro que como METRO 0 , β 2 1 y por lo tanto v C . ¿Lo que da?

Tenga en cuenta que el tiempo de evaporación t mi v

t mi v = 5120 π GRAMO 2 METRO 0 3 C 4
es finito Dado el tiempo suficiente, METRO 0 se irradia en su totalidad. Pero esto significaría que el agujero negro produciría una radiación masiva con v = C , que está prohibido. Si se emitieran fotones, entonces λ 0 , lo cual no tiene sentido. ¿Este problema tiene que ver con el hecho de que no sabemos cómo se comportaría la radiación de Hawking en la escala de Planck? ¿Esperaríamos que hubiera un límite inferior en la masa de los agujeros negros en una teoría cuántica de la gravedad?

Respuestas (1)

Hay una serie de problemas con su análisis.

La primera que me viene a la mente es que mantiene fija la masa de las partículas que se emiten. Claramente este no puede ser el caso: en algún momento la masa del agujero negro será menor que cualquier fijo metro .

Su suposición de que norte A parece totalmente infundado. De hecho, la tasa de partículas que se crean aumenta muy obviamente con la disminución A , por lo que esa suposición es claramente incorrecta.

Tiene sentido. Sin embargo, si consideras solo los fotones con energía mi = h F , usted obtiene PAG = h F ˙ , lo que da la ecuación C 6 15360 π GRAMO 2 METRO 2 = dieciséis π k GRAMO 2 METRO 2 C 4 h F lo que sugiere que como METRO 0 , λ 0
Estás haciendo suposiciones extrañas sobre la continuidad de norte también. Claramente en algún momento METRO C 2 < h F para la frecuencia que predice su fórmula. Tampoco entiendo tu justificación de por qué piensas norte A . Eso me parece totalmente infundado.
No se puede usar una métrica estática como la de Schwartzschild para modelar un agujero negro que también pierde masa rápidamente.
Por curiosidad, ¿qué métrica se debe utilizar?